Difference between revisions of "2020 AMC 8 Problems/Problem 4"

(Video Solution)
m (Undo revision 163507 by Eaglegeorge (talk) Was the previous edit a misclick?)
(Tag: Undo)
(33 intermediate revisions by 10 users not shown)
Line 1: Line 1:
 
==Problem==
 
==Problem==
 
Three hexagons of increasing size are shown below. Suppose the dot pattern continues so that each successive hexagon contains one more band of dots. How many dots are in the next hexagon?
 
Three hexagons of increasing size are shown below. Suppose the dot pattern continues so that each successive hexagon contains one more band of dots. How many dots are in the next hexagon?
 +
 
<asy>
 
<asy>
 +
// diagram by SirCalcsALot, edited by MRENTHUSIASM
 
size(250);
 
size(250);
real side1 = 1.5;
+
path p = scale(0.8)*unitcircle;
real side2 = 4.0;
+
pair[] A;
real side3 = 6.5;
 
real pos = 2.5;
 
pair s1 = (-10,-2.19);
 
pair s2 = (15,2.19);
 
 
pen grey1 = rgb(100/256, 100/256, 100/256);
 
pen grey1 = rgb(100/256, 100/256, 100/256);
 
pen grey2 = rgb(183/256, 183/256, 183/256);
 
pen grey2 = rgb(183/256, 183/256, 183/256);
fill(circle(origin + s1, 1), grey1);
+
for (int i=0; i<7; ++i) { A[i] = rotate(60*i)*(1,0);}
for (int i = 0; i < 6; ++i) {
+
path hex = A[0]--A[1]--A[2]--A[3]--A[4]--A[5]--cycle;
draw(side1*dir(60*i)+s1--side1*dir(60*i-60)+s1,linewidth(1.25));
+
fill(p,grey1);
}
+
draw(scale(1.25)*hex,black+linewidth(1.25));
fill(circle(origin, 1), grey1);
+
pair S = 6A[0]+2A[1];
for (int i = 0; i < 6; ++i) {
+
fill(shift(S)*p,grey1);
fill(circle(pos*dir(60*i),1), grey2);
+
for (int i=0; i<6; ++i) { fill(shift(S+2*A[i])*p,grey2);}
draw(side2*dir(60*i)--side2*dir(60*i-60),linewidth(1.25));
+
draw(shift(S)*scale(3.25)*hex,black+linewidth(1.25));
}
+
pair T = 16A[0]+4A[1];
fill(circle(origin+s2, 1), grey1);
+
fill(shift(T)*p,grey1);
for (int i = 0; i < 6; ++i) {
+
for (int i=0; i<6; ++i) {  
fill(circle(pos*dir(60*i)+s2,1), grey2);
+
fill(shift(T+2*A[i])*p,grey2);
fill(circle(2*pos*dir(60*i)+s2,1), grey1);
+
fill(shift(T+4*A[i])*p,grey1);
fill(circle(sqrt(3)*pos*dir(60*i+30)+s2,1), grey1);
+
fill(shift(T+2*A[i]+2*A[i+1])*p,grey1);
draw(side3*dir(60*i)+s2--side3*dir(60*i-60)+s2,linewidth(1.25));
 
 
}
 
}
 +
draw(shift(T)*scale(5.25)*hex,black+linewidth(1.25));
 
</asy>
 
</asy>
  
 
<math>\textbf{(A) }35 \qquad \textbf{(B) }37 \qquad \textbf{(C) }39 \qquad \textbf{(D) }43 \qquad \textbf{(E) }49</math>
 
<math>\textbf{(A) }35 \qquad \textbf{(B) }37 \qquad \textbf{(C) }39 \qquad \textbf{(D) }43 \qquad \textbf{(E) }49</math>
  
==Solution 1==
+
==Solution 1 (Pattern of the Rows)==
  
Looking at the rows of each hexagon, we see that the first hexagon has <math>1</math> dot, the second has <math>2+3+2</math> dots and the third has <math>3+4+5+4+3</math> dots, and given the way the hexagons are constructed, it is clear that this pattern continues. Hence the fourth hexagon has <math>4+5+6+7+6+5+4=\boxed{\textbf{(B) }37}</math> dots.
+
Looking at the rows of each hexagon, we see that the first hexagon has <math>1</math> dot, the second has <math>2+3+2</math> dots, and the third has <math>3+4+5+4+3</math> dots. Given the way the hexagons are constructed, it is clear that this pattern continues. Hence, the fourth hexagon has <math>4+5+6+7+6+5+4=\boxed{\textbf{(B) }37}</math> dots.
  
==Solution 2==
+
==Solution 2 (Pattern of the Bands)==
The first hexagon has <math>1</math> dot, the second hexagon has <math>1+6</math> dots, the third hexagon <math>1+6+12</math> dots, and so on. The pattern continues since to go from hexagon <math>n</math> to hexagon <math>(n+1)</math>, we add a new ring of hexagons around the outside of the existing ones, with each side of the ring having side length <math>(n+1)</math>. Thus the number of hexagons added is <math>6(n+1)-6 = 6n</math> (we subtract <math>6</math> as each of the corner hexagons in the ring is counted as part of two sides), confirming the pattern. We therefore predict that that the fourth hexagon has <math>1+6+12+18=\boxed{\textbf{(B) }37}</math> dots.
+
The first hexagon has <math>1</math> dot, the second hexagon has <math>1+6</math> dots, the third hexagon <math>1+6+12</math> dots, and so on. The pattern continues since to go from hexagon <math>n</math> to hexagon <math>(n+1),</math> we add a new band of dots around the outside of the existing ones, with each side of the band having side length <math>(n+1).</math> Thus the number of dots added is <math>6(n+1)-6 = 6n</math> (we subtract <math>6</math> as each of the corner hexagons in the band is counted as part of two sides), confirming the pattern. We therefore predict that that the fourth hexagon has <math>1+6+12+18=\boxed{\textbf{(B) }37}</math> dots.
  
==Solution 3 (variant of Solution 2)==
+
== Solution 3 (Variant of Solution 2) ==
 +
The dots in the next hexagon have four bands. From innermost to outermost:
 +
<ol style="margin-left: 1.5em;">
 +
  <li>The first band has <math>1</math> dot.</li><p>
 +
  <li>The second band has <math>6</math> dots: <math>1</math> dot at each vertex of the hexagon.</li><p>
 +
  <li>The third band has <math>6+6\cdot1=12</math> dots: <math>1</math> dot at each vertex of the hexagon and <math>1</math> other dot on each edge of the hexagon.</li><p>
 +
  <li>The fourth band has <math>6+6\cdot2=18</math> dots: <math>1</math> dot at each vertex of the hexagon and <math>2</math> other dots on each edge of the hexagon.</li><p>
 +
</ol>
 +
Together, the answer is <math>1+6+12+18=\boxed{\textbf{(B) }37}.</math>
  
Let the number of dots in the first hexagon be <math>h_0 = 1</math>. By the same argument as in Solution 2, we have <math>h_n=h_{n-1}+6n</math> for <math>n > 0</math>. Using this, we find that <math>h_1=7</math>, <math>h_2=19,</math> and <math>h_3=\boxed{\textbf{(B) }37}</math>.
+
~MRENTHUSIASM
  
==Video Solution==
+
==Solution 4 (Variant of Solution 2)==
https://youtu.be/SPNobOd4t1c (Channel also has resources to prepare for your AIME qualification)
+
 
 +
Let the number of dots in the first hexagon be <math>h_0 = 1.</math> By the same argument as in Solution 2, we have <math>h_n=h_{n-1}+6n</math> for <math>n > 0.</math> Using this, we find that <math>h_1=7,h_2=19,</math> and <math>h_3=\boxed{\textbf{(B) }37}.</math>
 +
 
 +
== Solution 5 (Brute Force) ==
 +
From the full diagram below, the answer is <math>\boxed{\textbf{(B) }37}.</math>
 +
<asy>
 +
// diagram by SirCalcsALot, edited by MRENTHUSIASM
 +
size(400);
 +
path p = scale(0.8)*unitcircle;
 +
pair[] A;
 +
pen grey1 = rgb(100/256, 100/256, 100/256);
 +
pen grey2 = rgb(183/256, 183/256, 183/256);
 +
for (int i=0; i<7; ++i) { A[i] = rotate(60*i)*(1,0);}
 +
path hex = A[0]--A[1]--A[2]--A[3]--A[4]--A[5]--cycle;
 +
fill(p,grey1);
 +
draw(scale(1.25)*hex,black+linewidth(1.25));
 +
pair S = 6A[0]+2A[1];
 +
fill(shift(S)*p,grey1);
 +
for (int i=0; i<6; ++i) { fill(shift(S+2*A[i])*p,grey2);}
 +
draw(shift(S)*scale(3.25)*hex,black+linewidth(1.25));
 +
pair T = 16A[0]+4A[1];
 +
fill(shift(T)*p,grey1);
 +
for (int i=0; i<6; ++i) {
 +
fill(shift(T+2*A[i])*p,grey2);
 +
fill(shift(T+4*A[i])*p,grey1);
 +
fill(shift(T+2*A[i]+2*A[i+1])*p,grey1);
 +
}
 +
draw(shift(T)*scale(5.25)*hex,black+linewidth(1.25));
 +
 
 +
pair R = 30A[0]+6A[1];
 +
fill(shift(R)*p,grey1);
 +
for (int i=0; i<6; ++i) {
 +
fill(shift(R+2*A[i])*p,grey2);
 +
fill(shift(R+4*A[i])*p,grey1);
 +
fill(shift(R+2*A[i]+2*A[i+1])*p,grey1);
 +
fill(shift(R+6*A[i+1])*p,grey2);
 +
fill(shift(R+2*A[i]+4*A[i+1])*p,grey2);
 +
fill(shift(R+4*A[i]+2*A[i+1])*p,grey2);
 +
}
 +
draw(shift(R)*scale(7.25)*hex,black+linewidth(1.25));
 +
</asy>
 +
~MRENTHUSIASM
 +
 
 +
== Video Solution ==
 +
https://www.youtube.com/watch?v=_IjQnXnVKeU
  
 +
==Video Solution by WhyMath==
 +
https://youtu.be/szWgrOPNw8c
  
 +
~savannahsolver
  
 +
==Video Solution==
 
https://youtu.be/eSxzI8P9_h8
 
https://youtu.be/eSxzI8P9_h8
 +
 +
==Video Solution by Interstigation==
 +
https://youtu.be/YnwkBZTv5Fw?t=123
 +
 +
~Interstigation
  
 
==See also==  
 
==See also==  
 
{{AMC8 box|year=2020|num-b=3|num-a=5}}
 
{{AMC8 box|year=2020|num-b=3|num-a=5}}
 
{{MAA Notice}}
 
{{MAA Notice}}

Revision as of 19:02, 14 October 2021

Problem

Three hexagons of increasing size are shown below. Suppose the dot pattern continues so that each successive hexagon contains one more band of dots. How many dots are in the next hexagon?

[asy] // diagram by SirCalcsALot, edited by MRENTHUSIASM size(250); path p = scale(0.8)*unitcircle; pair[] A; pen grey1 = rgb(100/256, 100/256, 100/256); pen grey2 = rgb(183/256, 183/256, 183/256); for (int i=0; i<7; ++i) { A[i] = rotate(60*i)*(1,0);} path hex = A[0]--A[1]--A[2]--A[3]--A[4]--A[5]--cycle; fill(p,grey1); draw(scale(1.25)*hex,black+linewidth(1.25)); pair S = 6A[0]+2A[1]; fill(shift(S)*p,grey1); for (int i=0; i<6; ++i) { fill(shift(S+2*A[i])*p,grey2);} draw(shift(S)*scale(3.25)*hex,black+linewidth(1.25)); pair T = 16A[0]+4A[1]; fill(shift(T)*p,grey1); for (int i=0; i<6; ++i) {   fill(shift(T+2*A[i])*p,grey2);  fill(shift(T+4*A[i])*p,grey1);  fill(shift(T+2*A[i]+2*A[i+1])*p,grey1); } draw(shift(T)*scale(5.25)*hex,black+linewidth(1.25)); [/asy]

$\textbf{(A) }35 \qquad \textbf{(B) }37 \qquad \textbf{(C) }39 \qquad \textbf{(D) }43 \qquad \textbf{(E) }49$

Solution 1 (Pattern of the Rows)

Looking at the rows of each hexagon, we see that the first hexagon has $1$ dot, the second has $2+3+2$ dots, and the third has $3+4+5+4+3$ dots. Given the way the hexagons are constructed, it is clear that this pattern continues. Hence, the fourth hexagon has $4+5+6+7+6+5+4=\boxed{\textbf{(B) }37}$ dots.

Solution 2 (Pattern of the Bands)

The first hexagon has $1$ dot, the second hexagon has $1+6$ dots, the third hexagon $1+6+12$ dots, and so on. The pattern continues since to go from hexagon $n$ to hexagon $(n+1),$ we add a new band of dots around the outside of the existing ones, with each side of the band having side length $(n+1).$ Thus the number of dots added is $6(n+1)-6 = 6n$ (we subtract $6$ as each of the corner hexagons in the band is counted as part of two sides), confirming the pattern. We therefore predict that that the fourth hexagon has $1+6+12+18=\boxed{\textbf{(B) }37}$ dots.

Solution 3 (Variant of Solution 2)

The dots in the next hexagon have four bands. From innermost to outermost:

  1. The first band has $1$ dot.
  2. The second band has $6$ dots: $1$ dot at each vertex of the hexagon.
  3. The third band has $6+6\cdot1=12$ dots: $1$ dot at each vertex of the hexagon and $1$ other dot on each edge of the hexagon.
  4. The fourth band has $6+6\cdot2=18$ dots: $1$ dot at each vertex of the hexagon and $2$ other dots on each edge of the hexagon.

Together, the answer is $1+6+12+18=\boxed{\textbf{(B) }37}.$

~MRENTHUSIASM

Solution 4 (Variant of Solution 2)

Let the number of dots in the first hexagon be $h_0 = 1.$ By the same argument as in Solution 2, we have $h_n=h_{n-1}+6n$ for $n > 0.$ Using this, we find that $h_1=7,h_2=19,$ and $h_3=\boxed{\textbf{(B) }37}.$

Solution 5 (Brute Force)

From the full diagram below, the answer is $\boxed{\textbf{(B) }37}.$ [asy] // diagram by SirCalcsALot, edited by MRENTHUSIASM size(400); path p = scale(0.8)*unitcircle; pair[] A; pen grey1 = rgb(100/256, 100/256, 100/256); pen grey2 = rgb(183/256, 183/256, 183/256); for (int i=0; i<7; ++i) { A[i] = rotate(60*i)*(1,0);} path hex = A[0]--A[1]--A[2]--A[3]--A[4]--A[5]--cycle; fill(p,grey1); draw(scale(1.25)*hex,black+linewidth(1.25)); pair S = 6A[0]+2A[1]; fill(shift(S)*p,grey1); for (int i=0; i<6; ++i) { fill(shift(S+2*A[i])*p,grey2);} draw(shift(S)*scale(3.25)*hex,black+linewidth(1.25)); pair T = 16A[0]+4A[1]; fill(shift(T)*p,grey1); for (int i=0; i<6; ++i) {   fill(shift(T+2*A[i])*p,grey2);  fill(shift(T+4*A[i])*p,grey1);  fill(shift(T+2*A[i]+2*A[i+1])*p,grey1); } draw(shift(T)*scale(5.25)*hex,black+linewidth(1.25));  pair R = 30A[0]+6A[1]; fill(shift(R)*p,grey1); for (int i=0; i<6; ++i) {   fill(shift(R+2*A[i])*p,grey2);  fill(shift(R+4*A[i])*p,grey1);  fill(shift(R+2*A[i]+2*A[i+1])*p,grey1);  fill(shift(R+6*A[i+1])*p,grey2);  fill(shift(R+2*A[i]+4*A[i+1])*p,grey2);  fill(shift(R+4*A[i]+2*A[i+1])*p,grey2); } draw(shift(R)*scale(7.25)*hex,black+linewidth(1.25)); [/asy] ~MRENTHUSIASM

Video Solution

https://www.youtube.com/watch?v=_IjQnXnVKeU

Video Solution by WhyMath

https://youtu.be/szWgrOPNw8c

~savannahsolver

Video Solution

https://youtu.be/eSxzI8P9_h8

Video Solution by Interstigation

https://youtu.be/YnwkBZTv5Fw?t=123

~Interstigation

See also

2020 AMC 8 (ProblemsAnswer KeyResources)
Preceded by
Problem 3
Followed by
Problem 5
1 2 3 4 5 6 7 8 9 10 11 12 13 14 15 16 17 18 19 20 21 22 23 24 25
All AJHSME/AMC 8 Problems and Solutions

The problems on this page are copyrighted by the Mathematical Association of America's American Mathematics Competitions. AMC logo.png